Solving Eigenvalue Questions for 2x2 Matrix & nxn Matrix

  • Thread starter Thread starter buzzmath
  • Start date Start date
  • Tags Tags
    Eigenvalue Matrix
Click For Summary
The discussion centers on solving eigenvalue problems for both a 2x2 matrix and an nxn matrix. For the 2x2 matrix, the user is working to prove that L1^2 + L2^2 is less than or equal to a^2 + b^2 + c^2 + d^2, and has made progress by relating the determinant and trace to the matrix entries. They need to show that 2bc is less than or equal to b^2 + c^2, with equality when b equals c. In the second question regarding the nxn matrix, the user is exploring the implications of positive eigenvectors and the condition that the sum of each row equals 1, questioning if the associated eigenvalue remains less than or equal to 1 without the positivity constraint. They seek clarification on their reasoning and completeness of their arguments for both problems.
buzzmath
Messages
108
Reaction score
0
I have two questions

1. If I have a 2x2 matrix A with entries a, b, c, d where a is the upper left corner, b upper right corner, c lower left, and d lower right. I have eigenvalues L1 and L2. I need to show that L1^2 + L2^2 <= a^2 + b^2 + c^2 + d^2. So far I've done this: I know det(A)=L1*L2 and that tr(A) = L1 + L2. So L1 + L2 = a + b square each side and get L1^2 + L2^2 +2*L1*L2 = a^2 + d^2 +2ad subtract 2*L1*L2 from each side and get L1^2 + L2^2 = a^2+d^2+2ad-2*L1*L2 where since L1*L2 is det(A) I have a^2+d^2 + 2ad-2ad+2bc thus L1^2 + L2^2 = a^2 + d^2 + 2bc. Now all I would really need to show is that 2bc <= b^2+c^2. I also need to know when this inequality is equal. Am I on the right track or does anyone have any advice?

2.If A is an nxn matrix where the sum of each row is 1 and all entires are positive. if v is an eigenvector of A with positive componentsShow the the associated eigenvalue is less than are equal to 1. Also, if we drop the requirement that the components of the eigenvector v be positive, is it still true that the associated eigenvalue is less than or equal to 1 in absolute value terms? Justify your answer. For this problem I was thinking I could consider the largest entry of v and take the corresponding entry in A. the largest that this A value could be is 1. If the value in v is less than 1 then the eigenvalue is then the most the eigenvalue could be is 1. If the value in v is 1 then the eigen value could be at most whatever the corresponding term in A is. If the value in v is greater than 1 then the eigenvalue is 1. I think I might be on the right track but I'm not really sure if this is a real clear/valid argument or if it's complete or how to put it together so it sounds lilke a clear proof. For the secone part I'm not really sure how to do it. Does anyone have any advice/help.

Thanks
 
Physics news on Phys.org
1. Very good so far. For any b, c, (b-c)^2= b^2- 2bc+ c^2\ge 0 so b^2+ c^2\ge 2bc. Of course, equality holds when b= c.
 
Thanks, I think I got the first one now. Any advice on the second one? Am I on the right track?
 
Question: A clock's minute hand has length 4 and its hour hand has length 3. What is the distance between the tips at the moment when it is increasing most rapidly?(Putnam Exam Question) Answer: Making assumption that both the hands moves at constant angular velocities, the answer is ## \sqrt{7} .## But don't you think this assumption is somewhat doubtful and wrong?

Similar threads

Replies
2
Views
2K
  • · Replies 8 ·
Replies
8
Views
2K
  • · Replies 3 ·
Replies
3
Views
3K
  • · Replies 4 ·
Replies
4
Views
1K
Replies
8
Views
1K
  • · Replies 19 ·
Replies
19
Views
4K
  • · Replies 2 ·
Replies
2
Views
2K
Replies
8
Views
3K
  • · Replies 2 ·
Replies
2
Views
1K
  • · Replies 5 ·
Replies
5
Views
2K